Talk:Population dynamics: Difference between revisions

From Puella Magi Wiki
Jump to navigation Jump to search
(Created page with "== refined model analysis == Based off one of my courses, I made a quick analysis of the refined model in terms of convergence and equilibrium. The theorems used are [citation n...")
 
mNo edit summary
Line 32: Line 32:


'''TODO'''. I'll add it but my brain just melt due to overheating. Haven't finished one of the cases and haven't checked that the results are coherent
'''TODO'''. I'll add it but my brain just melt due to overheating. Haven't finished one of the cases and haven't checked that the results are coherent
On a totally unrelated note, >mfw more than 350 tweets about this page in the past hour, and 20-50 more every minute...

Revision as of 13:01, 19 March 2011

refined model analysis

Based off one of my courses, I made a quick analysis of the refined model in terms of convergence and equilibrium. The theorems used are [citation needed] though since they're directly taken from said course's notes (and I only have references for the whole course). Plus I'm not sure the vocabulary is correct in English (translated from french). Please correct it if necessary.

Here goes:

Let's rename the variables for easier notation. x1 will be M(t) and x2 will be W(t). We can generalize the system as follows:

Population analysis, equation 1.png

Where a_ij is the contribution of population j to population i (so a12 is the effect witches have on magical girls, etc)

We can write this system using matrix notation:

Population dynamics, matrix notation.png

Where A and C are matrices containing the coefficients. To be exact, let A be

Population dynamics matrix A.png

And let C be

File:Population dynamics matrix C.png

The matrix C is constant and hence does not influence the results on stability/equilibrium.

We can prove that (first citation needed), it A's determinant is different than zero, then the only equilibrium for both M(t) and W(t) is when M(0) = 0 and W(0) = 0. (see observations on the page). The determinant of the matrix A is

Dynamics matrix5.png

Let us now discuss the stability, and other equilibria when det A yelds zero (i.e. B=K or D=-B)

TODO. I'll add it but my brain just melt due to overheating. Haven't finished one of the cases and haven't checked that the results are coherent

On a totally unrelated note, >mfw more than 350 tweets about this page in the past hour, and 20-50 more every minute...